please help solve this

Please Help Solve This

Answers

Answer 1

Answer:

\((-1, -16)\)

Step-by-step explanation:

First, we can expand the right side of this equation.

\(y=(x + 5)(x-3)\)

\(y = x^2 -3x + 5x - 15\)

\(y = x^2 + 2x - 15\)

Then, we can move the 15 to the left side.

\(y + 15 = x^2 + 2x\)

Now, the equation is in a form where we can complete the square for x. Remember that anything we add to one side, we also have to add to the other side.

\(y + 15 + (2/2)^2 = x^2 + 2x + (2/2)^2\)

\(y + 16 = x^2 + 2x + 1\)

\(y + 16 = (x+1)^2\)

Finally, we can move the 16 to the right side.

\(\boxed{y = (x+1)^2 - 16}\)

The equation of the parabola is now in vertex form:

\(y = (x - h)^2 + k\),

where \((h,k)\) is the vertex of the parabola.

In the equation we just solved for (\(y = (x+1)^2 - 16\)), we can identify the following:

\(h = -1\)\(k = -16\)

Therefore, the vertex of the parabola is:

\(\boxed{(-1, -16)}\)


Related Questions

Select each angle of rotation about the origin, (0, 0) that maps WXYZ TO W’ X’ Y’Z

Select each angle of rotation about the origin, (0, 0) that maps WXYZ TO W X YZ

Answers

The angle of rotation about the origin, (0, 0) that maps WXYZ TO W’ X’ Y’Z are

180 degrees

540 degrees

900 degrees

How to find the angle of rotation

The transformation rule for an angle of 180 degrees is simply a reflection or a flip. This means that any object or figure that undergoes this transformation will be flipped over a line or axis.

The rule is (x, y) will become (-x, -y)

Where 540 degrees = 360 + 180

where 900 degrees = 360 + 360 + 180

Learn more about angle of rotation at

https://brainly.com/question/28672446

#SPJ1

Solve for x. Round to the nearest tenth, if necessary.
56
Z
43°
x
M

Solve for x. Round to the nearest tenth, if necessary.56Z43xM

Answers

60.2 is the value of x in Trigonometric ratios

What are the trigonometric ratios?

Trigonometric ratios are used to describe the side ratios of a right triangle. The sine (sin), the cosine (cos), and the tangent are three popular trigonometric ratios (tan).

                      Greek is the language of measuring, "metric," and "trigon" is the language of triangles. It is possible to measure a right triangle specifically using its trigonometric ratios.

43 degree as reference angle

using tan rule

tan 43 = opposite / adjacent

0.93 = x / 56

x = 56/0.93

x = 60.21

x = 60.2

Learn more about trigonometric ratios

brainly.com/question/25122825

#SPJ1

help here's a pic on the problem

help here's a pic on the problem

Answers

The linear function that shows the proportional relationship between the distance and time is d = 100t

Linear Function

A linear function is a function that represents a straight line on the coordinate plane. For example, y = 3x - 2 represents a straight line on a coordinate plane and hence it represents a linear function. Since y can be replaced with f(x), this function can be written as f(x) = 3x - 2.

A linear function is of the form f(x) = mx + b where 'm' and 'b' are real numbers. Isn't it looking like the slope-intercept form of a line which is expressed as y = mx + b? Yes, this is because a linear function represents a line, i.e., its graph is a line. Here,

'm' is the slope of the line'b' is the y-intercept of the line'x' is the independent variable'y' (or f(x)) is the dependent variable

From the table given, we can find the slope of the function by using the formula

m = y₂ - y₁ / x₂ - x₁

m = 300 - 200 / 3 - 2

m = 100

To find the y - intercept,

y = mx + c

300 = 100(3) + c

300 = 300 + c

c = 300 - 300

c = 0

The equation that shows the relationship is

d = 100t

Learn more on linear function here;

https://brainly.com/question/15602982

#SPJ1

Is there a proportional relationship between number of sandwiches and the cost of the order? Explain how you know.


(In the picture are my answers so you guys can see them, but what I need to know is if they are proportional or not, and why.)

(20 points)

Is there a proportional relationship between number of sandwiches and the cost of the order? Explain

Answers

Answer:

CMER BOOOOOOOOOOOOOOOOOOOOOO

Step-by-step explanation:

Select the number that round to 387.4 when rounded to the nearest tenth.
A. 387.461
B. 387.344
C. 387.309
D. 387.352
E. 387.779

Answers

Answer:

D

Step-by-step explanation:

When rounding, the number 5 is rounded up. So the number 387.352 will be rounded up 387.4. Other options are not suitable. Correct answer is "D"

If you think my answer is the best, please mark it as the Brainliest.
Thank you! :))

AB is the line segment, M is the midpoint of AB. A is (-4,5) and M is (5, -1), what are the coordinates of B?

Answers

\(~~~~~~~~~~~~\textit{middle point of 2 points } \\\\ A(\stackrel{x_1}{-4}~,~\stackrel{y_1}{5})\qquad B(\stackrel{x_2}{x}~,~\stackrel{y_2}{y}) \qquad \left(\cfrac{ x_2 + x_1}{2}~~~ ,~~~ \cfrac{ y_2 + y_1}{2} \right) \\\\\\ \left(\cfrac{ x -4}{2}~~~ ,~~~ \cfrac{ y +5}{2} \right) ~~ = ~~\stackrel{\textit{\LARGE M} }{(5~~,~~-1)} \\\\[-0.35em] ~\dotfill\\\\ \cfrac{ x -4 }{2}=5\implies x-4=10\implies \boxed{x=14} \\\\[-0.35em] ~\dotfill\\\\ \cfrac{ y +5 }{2}=-1\implies y+5=-2\implies \boxed{y=-7}\)

7. Suppose random samples of 48 are taken from a distribution that is skewed right. The sampling distribution will be which of the following?

Answer choices:
Skewed left
Skewed right
Non-normal
Normal

Answers

Answer:

Step-by-step explanation:

The Central Limit Theorem states that as sample size increases, the sampling distribution of the sample mean becomes approximately normal, regardless of the shape of the population distribution, as long as the sample size is sufficiently large (usually n > 30).

In this case, the sample size is 48, which is larger than 30, so we can expect the sampling distribution of the sample mean to be approximately normal, even if the population distribution is skewed right. Therefore, the answer is "Normal".

Due in a MINUTE!!! Need help!

Due in a MINUTE!!! Need help!

Answers

Answer:

GH = 15

Step-by-step explanation:

In a trapezoid, the length of the median is one-half the sum of the lengths of the bases. Therefore:

\(9x - 3 = \frac{1}{2} (19 + 5x + 1)\)

\(18x - 6 = 5x + 20\)

\(13x = 26\)

\(x = 2\)

\(9(2) - 3 = 18 - 3 = 15\)

tony hit a baseball into the air with an initial upward velocity of 48 feet per second. The height h in feet of the ball above the ground can be modeled by h=-16t^2+48t+2. find the time it takes the ball to reach 38 feet above the ground

tony hit a baseball into the air with an initial upward velocity of 48 feet per second. The height h

Answers

Answer:

H=-6t^2+38t+1

Step-by-step explanation:

rotate the figure 90 degrees clockwise then translate 4 units left

rotate the figure 90 degrees clockwise then translate 4 units left

Answers

Answer:

If I am not mistaken, the new figure will be here (view attachment, bright blue figure is the new transformation)

rotate the figure 90 degrees clockwise then translate 4 units left

Answer:

For each of the following equations determine the output values Corresponding to the input value

(-2;-1;0;1;2;3)

Finn has $60 in a savings account. The interest rate is 10% per year and is not compounded. How much interest will he earn in 1 year?

Use the formula i = prt, where i is the interest earned, p is the principal (starting amount), r is the interest rate expressed as a decimal, and t is the time in years.

Answers

Answer:

6$

Step-by-step explanation:

Answer:

I = $ 6.00

Step-by-step explanation:

Equation:

I = Prt

Calculation:

First, converting R percent to r a decimal

r = R/100 = 10%/100 = 0.1 per year,

then, solving our equation

I = 60 × 0.1 × 1 = 6

I = $ 6.00

The simple interest accumulated

on a principal of $ 60.00

at a rate of 10% per year

for 1 years is $ 6.00.

Hope its correct

Mr garcia purchased a property 3 years ago for $155,750.00.The market has growth rate of 3.75% compounded monthly. What is the current value of the property?

Answers

Mr. Garcia purchased a property 3 years ago for $155,750.00

So, number of years = 3 and Initial amount = $155,750.00

The market has a growth rate of 3.75% compounded monthly.

So, interest rate = 3.75% and the number of times compounded in a year = 12

What is the current value of the property?​

Recall that the compound interest formula is given by

\(A=P(1+\frac{r}{n})^{nt}\)

Where

A = Future value (current value in this case)

P = Principle value (initial value = $155,750.00)

r = interest rate (growth rate in this case)

t = number of years (3 years in this case)

n = number of times compounding is done in a year (monthly means n = 12)

Let us substitute the given values into the above formula

\(\begin{gathered} A=155,750(1+\frac{0.0375}{12})^{12\cdot3} \\ A=155,750(1+0.003125)^{36} \\ A=155,750(1.003125)^{36} \\ A=155,750(1.11887) \\ A=\$174,264.00 \end{gathered}\)

Therefore, the current value of the property is $174,264.00

Solve for X. PLEASE HELP

Solve for X. PLEASE HELP

Answers

Answer:

xxtentacion for life

Step-by-step explanation:

X = 8


9 + 7 = 8 + x
16 = 8 + x
x = 8

3. What is the slope of the line through the points (2, 5) and (6, 13)?

Answers

Answer:

m = 2

Step-by-step explanation:

\(m=\frac{rise}{run}=\frac{13-5}{6-2}=\frac{8}{4}=\boxed{2}\)

Hope this helps.

6 * 10x^{-3}/8*10x^{-6}

Answers

The simplified form of the given expression, 6 × 10⁻³ / 8 × 10⁻⁶, is 3/4  × 10³

Simplifying an expression

From the question, we are to simplify the given expression

The given expression is

6 × 10⁻³ / 8 × 10⁻⁶

Simplifying the expression

6 × 10⁻³ / 8 × 10⁻⁶

This can be written as

6/8 × 10⁻³/10⁻⁶

Reduce the fraction

3/4  × 10⁻³/10⁻⁶

Apply the division law of indices

3/4  × 10⁻³ ⁻ ⁽⁻⁶⁾

3/4  × 10⁻³ ⁺ ⁶

3/4  × 10³

Hence, the value is 3/4  × 10³

Learn more on Simplifying an expression here: https://brainly.com/question/723406

#SPJ1

Each of the four sets a1, a2, a3, and a4 contains four elements. the intersection of every i of these four sets (2 < i < 4) consisters of 5 - i elements. What is a1 U a2 U a3 U a4?

Answers

Answer:

the new year was patrick's year and the second time she had the target audience in her career as an 34696468 )/684₩¥€=÷×+)^^*,?^&££)ktuioon

Quarters are currently minted with weights normally distributed and having a standard deviation of 0.065. New equipment is being tested in an attempt to improve quality by reducing variation. A simple random sample of 25 quarters is obtained from those manufactured with the new​ equipment, and this sample has a standard deviation of 0.047 . Use a 0.05 significance level to test the claim that quarters manufactured with the new equipment have weights with a standard deviation less than 0.065. Does the new equipment appear to be effective in reducing the variation of​ weights?

Answers

Answer:

Yes, the new equipment appear to be effective in reducing the variation of​ weights.

Step-by-step explanation:

We are given that Quarters are currently minted with weights normally distributed and having a standard deviation of 0.065.

A simple random sample of 25 quarters is obtained from those manufactured with the new​ equipment, and this sample has a standard deviation of 0.047.

Let \(\sigma\) = standard deviation of weights of new equipment.

SO, Null Hypothesis, \(H_0\) : \(\sigma \geq\) 0.065      {means that the new equipment have weights with a standard deviation more than or equal to 0.065}

Alternate Hypothesis, \(H_A\) : \(\sigma\) < 0.065      {means that the new equipment have weights with a standard deviation less than 0.065}

The test statistics that would be used here One-sample chi-square test statistics;

                           T.S. =  \(\frac{(n-1)s^{2} }{\sigma^{2} }\)  ~ \(\chi^{2}__n_-_1\)

where, s = sample standard deviation = 0.047

           n = sample of quarters = 25

So, the test statistics  =  \(\frac{(25-1)\times 0.047^{2} }{0.065^{2} }\)  ~  \(\chi^{2}__2_4\)   

                                     =  12.55

The value of chi-square test statistics is 12.55.

Now, at 0.05 significance level the chi-square table gives critical value of 13.85 at 24 degree of freedom for left-tailed test.

Since our test statistic is less than the critical value of chi-square as 12.55 < 13.85, so we have sufficient evidence to reject our null hypothesis as it will fall in the rejection region due to which we reject our null hypothesis.

Therefore, we conclude that the new equipment have weights with a standard deviation less than 0.065.

What is the answer of this triangle congruence question.

What is the answer of this triangle congruence question.

Answers

The value of x in the triangles are 9.

What is a quadratic equation?

For variable x : ax² + bx + c = 0, where a≠0 is a standard quadratic equation, which is a second-order polynomial equation in a single variable. It has at least one solution since it is a second-order polynomial equation, which is guaranteed by the algebraic basic theorem.

Given:

The triangles are congruent.

That means, their corresponding angles are also congruent.

In ΔJKL,

the sum of all the angles of the triangle is 180°.

So,

x²-2x + x + 29 + 3x + 52 = 180

x² + 2x - 99 = 0

Solving the quadratic equation,

x² +11x - 9x - 99 = 0.

x (x + 11) -9 (x + 11) = 0

x = 9 and x = -11

Here, we take x = 9.

Therefore, the value of x is 9.

To learn more about the quadratic equation;

https://brainly.com/question/17177510

#SPJ1


Jamie went out to her grandfather's farm.
Her grandfather has pigs and chickens on his farm.
She noticed that there were a total of 26 heads and
68 feet among them. How many chickens and how
many pigs did her grandfather have?

Answers

Answer:

8 pigs

Step-by-step explanation:

Since every animal has only 1 head, a total of 26 heads suggests that there are 26 animals in total.

Suppose all animals are chickens.

Total no. of feet = 26 chickens × 2 feet

                           = 52 feet

Difference in total no. of feet = 68 feet - 52 feet

                                                = 16 feet

Difference in no. of feet each animal has

= 4 feet (a pig) - 2 feet (a chicken)

= 2 feet

∴ No. of pigs = 16 feet ÷ 2 feet

                      = 8 pigs

The conditional statement below is true. If possible, write the biconditional statement.

If 2x = 18, then x = 9.

Answers

The biconditional statement for the given conditional statement would be:

2x = 18 if and only if x = 9.

The given conditional statement "If 2x = 18, then x = 9" can be represented symbolically as p → q, where p represents the statement "2x = 18" and q represents the statement "x = 9".

To form the biconditional statement, we need to determine if the converse of the conditional statement is also true. The converse of the original statement is "If x = 9, then 2x = 18". Let's evaluate the converse statement.

If x = 9, then substituting this value into the equation 2x = 18 gives us 2(9) = 18, which is indeed true. Therefore, the converse of the original statement is true.

Based on this, we can write the biconditional statement:

2x = 18 if and only if x = 9.

The biconditional statement implies that if 2x is equal to 18, then x must be equal to 9, and conversely, if x is equal to 9, then 2x is equal to 18. The biconditional statement asserts the equivalence between the two statements, indicating that they always hold true together.

In summary, the biconditional statement is a concise way of expressing that 2x = 18 if and only if x = 9, capturing the mutual implication between the two statements.

for such more question on conditional statement

https://brainly.com/question/27839142

#SPJ8

I need help again
I am
Sorry

I need help again I am Sorry

Answers

Answer:

x = 171.16 - 5.8

x = 165.36

please give me brainlist

Step-by-step explanation:

Answer:

165.36 is Pete's height

Step-by-step explanation

You have to subtract 5.8 by 171.16 and that will be Pete's height.

Solve the inequality for w.
w+7<20
Simplify your answer as much as possible.
0

Answers

Answer:

w<13

Step-by-step explanation:

Works identically to a normal single-variable equation.
Subtract 7 on both sides in order to isolate w--->w+7-7<20-7
The answer (which cannot be simplified any further) is w<13.

Answer:

w < 1`3

Step-by-step explanation:

Isolate the variable w on one side of the inequality sign.

w+7<20

w<20 - 7

w<13.

i
You are given the difference of the numbers of boys and girls in a class and the ratio of boys to girls. How many boys
and how many girls are in the class?
4 more girls; 9 for every 13
There are
boys and
girls in the class.

iYou are given the difference of the numbers of boys and girls in a class and the ratio of boys to girls.

Answers

Answer:

Maybe they are all 13s th am sure

Simplify. Write your answers without exponents.

Simplify. Write your answers without exponents.

Answers

Answer:

1st one is: 1/125

2nd one is: 1/16

0.008 is the first answer 0.0625 is the second answer

Find the slope of the line that contains the points (1, 6) and (10, -9)

Answers

Answer:

-15/9

Step-by-step explanation:

The slope is the steepness of the line. To find it, you divide how far up it goes by how far across it goes. This equals the diagonal incline. To find it with points, you find the difference between the y values and divide that by the difference in the x values!

-9-6=-15

10-1=9

-15/9=-15/9

This slope means that for every nine units the line goes across, it goes down 15 units!

Have a wonderful day :D

7. Alexis has 145 stickers. She wants to divide the stickers evenly among 5 of her friends. How
many stickers will each friend get?
Write your answer in the box.
stickers.
8. There are 2,472 seats in the school auditorium. The seats are divided into 6 equal sections.
How many seats are in each section? Write your answer in the box.
seats
Explain how you found your answer. Show your work.

Answers

Answer: 7: 29 stickers for each friend

8: 412 seats in each section

Step-by-step explanation:

We will have to divide 145 (stickers) by 5 (friends) to get 29

We will have to divide 2472 by 6 to get 412

7.) 145 ÷ 5 = 29 → Each friend will get 29 stickers

8.) 2,472 ÷ 6 = 412 → There are 412 seats in each second.

I divided for both problems using the long division method and multiplied which is the inverse operation, to check my work.

how do I solve this screen shot

how do I solve this screen shot

Answers

Answer:

add me as a friend and give me thanks

Step-by-step explanation:

the answer is A for sure

Simplify: 4 1/2
÷ 2:

Answers

Answer:

9/4 = 2 1/4

Step-by-step explanation:

Solve the system of equations by using elimination. Write the solution as an ordered pair.

4x−3y=35x+6y=7

Answers

Answer is (1, 1/3)

Step by step

Our equations are

4x -3y = 3 ➡️ equation 1

5x + 6y = 7 ➡️ equation 2

Using elimination method we need to change one equation so one variable can be eliminated and then solve

I will multiply all terms in equation 1 by 2

2 ( 4x -3y = 3)

8x -6y = 6 ➡️ equation 1

5x + 6y = 7 ➡️ equation 2

So you see we can eliminate -6y and +6y by adding the like terms in the equations

13x = 13

Divide both sides by 13 to solve for x

13/13 x = 13/13

x = 1

Now substitute the value of x = 1 into equation 2

5x + 6y = 7 ➡️ equation 2

5(1) + 6y = 7
5 + 6y = 7
Subtract 5 from both sides to isolate variable

5 -5 + 6y = 7 -5
Simplify

6y = 2

Divide both sides by 6 to solve for y

6/6 y = 2/6
Simplify

y = 1/3

I graphed it to check my work, solution is correct
Solve the system of equations by using elimination. Write the solution as an ordered pair. 4x3y=35x+6y=7

Write an equation of the line, in point-slope form, that passes through the two given points. points: (-3.18), (12.-12)​

Answers

Answer:

y=-2x+12

Step-by-step explanation:

Count how much the y raises for each x through division for the slop, and then plug that in to the x on either one of the pairs to get the number, and then add or subtract whatever the actual y is to get the y intercept.

Other Questions
Presented below is information from Vaughn Computers Incorporated.July 1Sold $24,700 of computers to Robertson Company with terms 3/15, n/60. Vaughn uses the gross method to record cash discounts. Vaughn estimates allowances of $1,341 will be honored on these sales.10Vaughn received payment from Robertson for the full amount owed from the July transactions.17Sold $207,200 in computers and peripherals to The Clark Store with terms of 2/10, n/30.30The Clark Store paid Vaughn for its purchase of July 17.Prepare the necessary journal entries for Vaughn Computers. (i tried to answer some of the problems on my own. If they are wrong, please answer this question with the new chart with new numbers and answers in the exact format below.) What is the process called to prepare a ceramic by heating a slurry of a powder of an inorganic substance in water to a very high temperature under pressure Which polynomial is prime? x2 x 1 x2 x 2 x2 12x 11 x2 2x 8. alin co. purchases a building for $300,000 and pays an additional $30,000 for title fees and lawyer fees. alin also pays $20,000 in renovations, including painting, carpet, lighting, etc. alin should record the cost of the building at: multiple choice question. $330,000. $320,000. $350,000. $300,000. Read the excerpt from Persepolis.What is the central idea of these panels?Marjane wants to be a prophet to help others face their fate.Marjane wants to be a prophet to make others wealthy.Marjane wants to be a prophet to abolish embarrassment.Marjane wants to be a prophet to fix the worlds injustices. Significant brain development of the _______ is finally completed during emerging adulthood. a 45-year-old man presents with episodic attacks of headache, recurring bouts of palpitations, anxiety, and sweating. he also gives history of a severe attack 1 week ago while he was having wine and cheese with his wife. on further questioning, he comments that he gets lightheaded when he stands up too rapidly. he comments that his mother had similar problems. on physical examination, his blood pressure is 165/90 mm hg and his heart rate is 80/min. a 24-hour collection of his urine tests positive for vanillylmandelic acid. imaging studies showed bilateral adrenal medullary hyperplasia. further workup showed hypercalcemia, hypophosphatemia, and increased parathyroid hormone levels. 5 friends walked into a restaurant. they all had a different birthday. there was a round table with 5 seats around it. what is the probability that they sit down in correct age order either clockwise or counter clockwise? 1. The table below represents a linear relationship. Find the slope of the line.Find the slope of each line that passes through the given points.2. (4,0) and (7,9)3. (9,2) and (6,4)4. (4, -1) and (4,3) in men, a waist-to-hip ratio higher than what is associated with an increased risk for chronic disease? Add heat until the lead melts. Does lead melt into individual atoms or molecules? An item is regularly priced at $15. Raina bought it at a discount of 65% off the regular price. How much did she pay "-24" 2 x - 10 < "-6" what term refers to cybersecurity competitions used to earn money and a reputation? Question 10 of 20What is a good question to ask yourself when choosing a career path?O A. What subjects do I like?B. What do my friends want to do?C. What do my parents want me to do?OD. What skills do I need to improve?SUBMIT Which is the coreect description of a revolution and one of the outcomes What details from the narration relate rainsfords impression of zaroff? select 4 options. almost bizarre quality about the general's face his thick eyebrows and pointed military mustache were as black as the night his eyes, too, were black and very bright the face of an aristocrat turning to the giant in uniform, the general made a sign the giant put away his pistol, saluted, withdrew. why do you think it is important to replace old or damaged cells? Describe the sequence of transformations that will carry ABC onto A1B1C1 Which rule describes the translation? (x, y) (x 8, y 3) (x, y) (x 3, y 8) (x, y) (x 8, y 3) (x, y) (x 3, y 8)